Hace unos años pregunté sobre la desigualdad Demostrar que $\int_0^\infty\frac1{x^x}\, dx<2$ . Cuando volví a revisarlo, encontré que cada una de las siguientes integrales de tetración $$\int_0^\infty\frac{dx}{x^x},\int_0^\infty\frac{dx}{x^{x^{x^x}}},\int_0^\infty\frac{dx}{x^{x^{x^{x^{x^x}}}}},\cdots$$ parecía estar delimitada por encima por $2$ . En el gráfico siguiente, cada índice representa la mitad del número de tetraciones.
Obviamente, un método de ataque consiste en demostrar que si $f_1(x)=x^x$ y $f_{k+1}(x)=x^{x^{f_k(x)}}$ entonces
-
$\int_0^\infty dx/f_{k+1}(x)>\int_0^\infty dx/f_k(x)$ para cada $k>1$ y
-
$\lim_{k\to\infty}\int_0^\infty dx/f_k(x)<2$ .
Observaciones.
- El primer paso del método anterior significa que el área ganada en el intervalo $(0,1)$ es mayor que el área perdida en $(1,\infty)$ . Parece que las diferencias consecutivas (representadas a continuación como
%97
) $$\int_0^\infty\frac{dx}{f_{k+1}(x)}-\int_0^\infty\frac{dx}{f_k(x)}$$ del orden de $k^{-\log k}$ y también disminuyen monotónicamente en la mayoría de los casos.
He publicado este problema de forma cruzada <a href="https://mathoverflow.net/questions/417026/is-int-0-inftydx-over-xxxx-int-0-inftydx-over-xxxxxx">en MathOverflow </a>.
- Para el segundo paso, el caso límite resulta muy fácil de demostrar. Tenemos $$\lim_{k\to\infty}\int_0^\infty\frac{dx}{f_k(x)}<\lim_{k\to\infty}\int_0^\infty\frac{dx}{g_k(x)}=-\int_0^\infty t\cdot\frac d{dt}\frac1{t^t}\,dt=\int_0^\infty\frac1{x^x}\,dx<2$$ donde $g_{k+1}(x)=x^{g_k(x)}$ y $g_1(x)=x$ .